Calculus 10th Edition

Published by Brooks Cole
ISBN 10: 1-28505-709-0
ISBN 13: 978-1-28505-709-5

Chapter P - Problem Solving - Page 40: 9

Answer

(a) $5$, greater (b) $3$, less (c) $4.1$, greater (d) $4+h$, yes; it is consistent with the above answers. (e) $m=4$. To find the tangent line at the given point, one can substitute the $"y"$ in $y=x^2$ with $"y=mx+d"$ and then force the resultant quadratic equation to have only one root (Please note that this method is not a general way to find tangent lines since there are many examples in which there exist lines passing through a point of the graph of a function while they are not tangent lines. However, if one applies this method and concludes the result which describes a unique line, then the method will be valid and the result will be correct). The general form of equation of line is $y=mx+d$. This line passes through the point $(2,4)$, so $$4=m \cdot 2 +d \quad \Rightarrow \quad d= 4-2m \,.$$Now we substitute$"y"$ in $y=x^2$ with $"y=mx+4-2m"$, so we have $$mx+4-2m=x^2 \quad \Rightarrow \quad x^2-mx+2m-4=0$$If we want that the above quadratic equation has only one root, its discriminant must be zero. Thus,$$\Delta=b^2-4ac=m^2-8m+16=0 \quad \Rightarrow \quad (m-4)^2=0$$ $$\Rightarrow \quad m=4.$$ One may expect the answer because of the above limiting behavior, which comes from some numerical computations. But keep in mind that this way of reasoning is not a correct and rigorous way of mathematical reasoning. There exist many examples that such numerical computations may mislead.

Work Step by Step

To find the slope of a line joining two points $(x_1,y_1)$ and $(x_2,y_2)$, one can use the formula $m=\frac{y_2-y_1}{x_2-x_1}$. The slope of the tangent line has been computed in the above answer (part (e)), and it is equal to $4$. (a)$$m=\frac{9-4}{3-2}=5>4$$ (b)$$m=\frac{1-4}{1-2}=3<4$$ (c)$$m=\frac{4.41-4}{2.1-2}=4.1>4$$ (d)$$\frac{(2+h)^2-4}{(2+h)-2}=\frac{4+4h+h^2-4}{h}=\frac{4h+h^2}{h}=4+h$$ (i) for (a), $h=1 \quad \Rightarrow4+h=4+1=5 \quad \checkmark$ (ii) for (b), $h=-1 \quad \Rightarrow 4+h=4-1=3 \quad \checkmark$ (iii) for (c), $h=0.1 \quad \Rightarrow 4+h= 4+0.1=4.1 \quad \checkmark$ (e) It has been explained in the answer.
Update this answer!

You can help us out by revising, improving and updating this answer.

Update this answer

After you claim an answer you’ll have 24 hours to send in a draft. An editor will review the submission and either publish your submission or provide feedback.